Difference between revisions of "2021 Fall AMC 12B Problems/Problem 4"
m |
Cellsecret (talk | contribs) (→Solution 2) |
||
Line 16: | Line 16: | ||
~NH14 | ~NH14 | ||
+ | ==Video Solution by Interstigation== | ||
+ | https://youtu.be/p9_RH4s-kBA?t=429 |
Revision as of 01:19, 24 November 2021
- The following problem is from both the 2021 Fall AMC 10B #5 and 2021 Fall AMC 12B #4, so both problems redirect to this page.
Problem
Let . Which of the following is equal to
Solution 1
We have Therefore,
~kingofpineapplz
Solution 2
The requested value is Thus, the answer is
~NH14